1989 AIME Problems/Problem 7

Revision as of 17:47, 28 February 2007 by Azjps (talk | contribs) (solution, box)

Problem

If the integer $k^{}_{}$ is added to each of the numbers $36^{}_{}$, $300^{}_{}$, and $596^{}_{}$, one obtains the squares of three consecutive terms of an arithmetic series. Find $k^{}_{}$.

Solution

Call the terms of the arithmetic progression $a,\ a + d,\ a + 2d$, making their squares $a^2,\ a^2 + 2ad + d^2,\ a^2 + 4ad + 4d^2$.

We know that $\displaystyle a^2 = 36 + k$ and $\displaystyle (a + d)^2 = 300 + k$, and subtracting these two we get $\displaystyle 264 = 2ad + d^2$ (1). Similarly, using $\displaystyle (a + d)^2 = 300 + k$ and $\displaystyle (a + 2d)^2 = 596 + k$, subtraction yields $\displaystyle 296 = 2ad + 3d^2$ (2).

Subtracting the first equation from the second, we get $\displaystyle 2d^2 = 32$, so $\displaystyle d = 4$. Substituting backwards yields that $\displaystyle a = 31$ and $\displaystyle k = 925$.

See also

1989 AIME (ProblemsAnswer KeyResources)
Preceded by
Problem 6
Followed by
Problem 8
1 2 3 4 5 6 7 8 9 10 11 12 13 14 15
All AIME Problems and Solutions